Đến nội dung

tuan101293 nội dung

Có 316 mục bởi tuan101293 (Tìm giới hạn từ 29-03-2020)



Sắp theo                Sắp xếp  

#458180 Chứng minh rằng $P.Q$ cũng có thể biếu diễn dưới dạng tổng của 4 số...

Đã gửi bởi tuan101293 on 17-10-2013 - 18:39 trong Số học

Bạn có thể tham khảo tại đây

http://en.wikipedia....square_identity

Thực tế bài toán là kết quả trực tiếp của đly lagrange ( mọi số nguyên dương đều là tổng của 4 số chính phương)




#456652 $\sum(\frac{ab}{3b+c+d+3} \leq frac...

Đã gửi bởi tuan101293 on 10-10-2013 - 20:27 trong Bất đẳng thức - Cực trị

Bài này ghép ghép vui phết

Nhân 2 vế với 9  và dùng bdt svac 

ta có $$\frac{9ab}{a+4b+2c+2d} \le \frac{ab}{a+2c} + \frac{2ab}{2b+d}$$

         $$\frac{9bc}{b+4c+2d+2a} \le \frac{2bc}{a+2c} +\frac{bc}{b+2d}$$

         $$\frac{9cd}{c+4d+2a+2b} \le \frac{2cd}{b+2d} +\frac{cd}{c+2a}$$

         $$\frac{9da}{d+4a+2b+2c} \le \frac{2da}{2a+c} +\frac{da}{d+2b}$$

Cộng dọc ta có $$9LHS \le a+b+c+d = 3$$

q,e,d




#456637 f(x+f(x+2y))=f(2x)+f(2y)

Đã gửi bởi tuan101293 on 10-10-2013 - 20:05 trong Phương trình hàm

Bạn ko hiểu chỗ nào ???

Lời giải bài 2: 

Ký hiệu P(x,y) là thay (x,y) vào đề bài: 

P(0,0) ta có $f(2) = 4$ 

P(0,y) ta có $ f(f(2y)) = f(2y) + 2$, từ biểu thức này thay liên tục y = 1,2,3,.... ta có $f(2n) = 2n+2$ với n dương bằng quy nạp

P(2k,-k) ta có $f(2k+2) = f(4k) +f(-2k) $ thay k =1,2,... ta tìm được nốt $f(2m) = 2m+2$ với m âm

hay ta luôn có $f(2n) = 2n+2$ với mọi n thuộc Z

Tiếp theo ta chứng minh

Nếu $a = b (mod 2)$ và $f(a) = f(b) $ thì $a = b$ (*)

Thật vậy, nếu $a = b( mod 2) $ và $f(a)  = f(b)$ ta có x,y,z nguyên  mà $x+2y = a, x+2z = b$

P(x,y) ta có $f(x+f(a)) = f(2x) +f(2y) = 2x+2y+4  $  (1)

P(x,z) ta có $f(x+f(b)) = f(2x) + f(2z) = 2x+2z+4$    (2) 

(1),(2) suy ra $y=z$ hay $a=b$ nên (*) đúng

Bây h ta tính f(1) 

TH1: f(1) lẻ, suy ra f(1)+1 chẵn và từ P(1,0) suy ra $f(1+f(1)) = 6 = f(4)$, theo (*) ta có $f(1) +1 =4$ hay f(1) = 3

TH2: f(1) chẵn hay $f(1) =2k$

P(-2k+1,k) ta có $f(-2k+1+ 2k) = -2k +6$ hay $f(1) = -2k+6$ suy ra $4k =6$ vô lý

hay f(1) =3

Cách tính f(2n+1) hoàn toàn tương tự

Tổng quát là $f(2m+1) = 2k$ thì thay P(2m-2k+1,k) cũng thấy vô lý.

Tóm lại f(x) = x+2

Ps: bài này mệt phết nhỉ @@




#455477 $\sqrt[3]{x-9}= (x-3)^3+6$

Đã gửi bởi tuan101293 on 05-10-2013 - 22:29 trong Phương trình - hệ phương trình - bất phương trình

viet lại pt $\sqrt[3]{x-9}+(x-9) = (x-3) + (x-3)^3$

nếu ta đặt $f(x) = x^3 + x$ (f là hàm tăng) ta có $f(\sqrt[3]{x-9}) = f(x-3)$

hay $\sqrt[3]{x-9} = x-3$, mũ 3 lên phân tích ta có x=1 và $x= 4+- i\sqrt{2}$




#455450 f(x+f(x+2y))=f(2x)+f(2y)

Đã gửi bởi tuan101293 on 05-10-2013 - 21:54 trong Phương trình hàm

Bài 2:

$f(f(n-1)) \ge 1 $ suy ra $f(n+1)\ge f(n)+1$ hay f là hàm tăng và $f(n)\ge n$

$f(n+1) =f(n) +f(f(n-1)) \ge f(n)+f(n-1) \ge 2n-1$ (1)

$f(f(n-1))< f(n+1)$ suy ra $f(n-1)<n+1$ hay $f(n)\le n+1$ với mọi n (2)

(1), (2) suy ra vô lý




#455443 Chứng minh tính chất: $a^{gcd(x,y)}\equiv b^{gcd(x,...

Đã gửi bởi tuan101293 on 05-10-2013 - 21:39 trong Số học

b nguyên tố cùng nhau với m nên tồn tại nghịch đảo mod m của b là c.

nhân $c^x$ vào pt 1, $c^y$ vào pt 2 ta có

$(ac)^x = 1 (mod m)$ và $(ac)^y = 1 (mod m)$

ac nguyên tố cùng nhau với m, nên q-cấp của ac mod m là ước của cả x và y hay $q|gcd(x,y)$

suy ra $(ac)^{gcd(x,y)} = 1$ mod m nhân lại b vào ta có dpcm




#454884 Cho $x+y+z=1$Chứng minh BĐT $\frac{x}{x+yz...

Đã gửi bởi tuan101293 on 03-10-2013 - 18:03 trong Bất đẳng thức và cực trị

$LHS = \sum \frac{x}{x(x+y+z)+yz}= \sum \frac{x}{(x+y)(x+z)}=\frac{\sum x(y+z)}{\prod (x+y)} =\frac{2(xy+yz+zx)}{\prod (x+y)}=\frac{2(xy+yz+zx)(x+y+z)}{\prod (x+y)}\le \frac{9}{4}$

ngược dấu bdt nhé




#454637 1 số bài toán vui

Đã gửi bởi tuan101293 on 02-10-2013 - 14:31 trong IQ và Toán thông minh

1, 3 người mà 2 người bất kỳ đều bị nhọ  thì cả 3 đều nhọ.

2, Nếu cô thứ 1 nói đúng suy ra cô 2 là Nhị và nói sai vào thứ 2, vô lý

    suy ra cô thứ 1 nói sai, cô thứ 1 là Nhị và ngày hôm đó là 3,5,7

   cô thứ 2 là Nhất, thứ 4 nói dối tức là cô ta cũng nói dối nên ngày hôm đó là thứ 3.

3, Cụ nói là: chỉ cần nhảy lên ngựa người kia và phi về đích là thắng mà. 




#454632 nghịch lý monty hall

Đã gửi bởi tuan101293 on 02-10-2013 - 14:13 trong IQ và Toán thông minh

cái này là conditional probability rõ ràng, sự kiện sau liên quan đến sự kiện trước, bạn rai_2601 nói không đúng nhé.

Cm thì có 1 bạn ở trên nói nghe 33+33=66 cũng đúng  nhưng để lập luận toán học thì người ta thường dùng Bayes theorem. có 1 dòng thôi à.

trước mình học cái này cũng hơi bất ngờ. Nhuwng hài ở chỗ là ai cũng nghĩ là 50% thành ra chơi ô cửa bí mật chả ai đổi =)).




#454629 CMR nếu $f(f(f(x)))=x, \forall x \in \mathbb{R}...

Đã gửi bởi tuan101293 on 02-10-2013 - 14:00 trong Phương trình hàm

Hoàn toàn sai nhé,

ví dụ $f(x) =\frac{x-3}{x+1} $




#454598 Tính giới hạn.

Đã gửi bởi tuan101293 on 02-10-2013 - 01:44 trong Giải tích

trong mục toán đại học thì bạn cứ khai triển taylor quanh lân cận 0 là ra hết mà (xài cái o(x) hay O(x)) đó 

ví dụ bài 1:

$e^x-e^{-x} = (1+x+\frac{x^2}{2}+\frac{x^3}{6}) - (1-x+\frac{x^2}{2}-\frac{x^3}{6})+o(x^3) = 2x+\frac{x^3}{3}+o(x^3)$

$sin(x) = x-\frac{x^3}{6}+o(x^3)$

nên $lim = 2$




#454597 Chứng minh rằng cấp của $s$ là $q^{a}$

Đã gửi bởi tuan101293 on 02-10-2013 - 01:38 trong Số học

$s^{q^\alpha} =1$ mod p 

suy ra cấp của s mod p là ước của $q^\alpha$ nên có dạng $q^t$ 

nếu $t\le \alpha -1$ thì$s^{q^t} = 1$ mod p , mũ q lên $\alpha -1-t$ lần ta có $x^{q^{\alpha-1}} = 1$ mod p vô lý nên $t=\alpha$ hay cấp =$q^\alpha$




#454596 $2n^{2}-1$

Đã gửi bởi tuan101293 on 02-10-2013 - 01:31 trong Số học

Bài toán vẫn đúng nếu với n bất kỳ, là 1 hệ quả của định lý Cauchy Davenport.

có thể tham khảo lời giải tại đây

http://www.tau.ac.il...a/PDFS/egz1.pdf




#454593 Cho n là số nguyên dương

Đã gửi bởi tuan101293 on 02-10-2013 - 00:18 trong Số học

ký hiệu $[n\sqrt{3}] = m$ và $a={n\sqrt{3}}=n\sqrt{3}-m$

ta có $3n^2-m^2>0$ nên $3n^2\ge m^2+2$ ($m^2+1$ không chia hết cho 3)

suy ra $3n^2-m^2\ge 2$ nên $a*(n\sqrt{3}+m) \ge 2$ suy ra $a\ge \frac{2}{n\sqrt{3}+m}>\frac{1}{n\sqrt{3}}$ 

q.e.d




#454583 $x^{4}-y^{4}=240$

Đã gửi bởi tuan101293 on 01-10-2013 - 23:11 trong Phương trình - Hệ phương trình - Bất phương trình

Bài này là bài 1 VMO 2010 

http://www.artofprob...533ba0#p2103782

ý tưởng là lấy pt trên trừ 8 pt dưới đề quy về bậc 4




#454393 $u_{1}=1;u_{2}=2;u_{n+1}=\dfrac{...

Đã gửi bởi tuan101293 on 01-10-2013 - 05:31 trong Dãy số - Giới hạn

Theo em thì có lẽ bài này sai đề, với lại nếu đúng đi nữa thì số hạng thứ 5 cũng không thuộc N nên có lẽ không cần quan tâm cho mệt @@




#454392 $f(f(x))=x \wedge f(f(x)+1)= 1- x $

Đã gửi bởi tuan101293 on 01-10-2013 - 04:37 trong Phương trình hàm

Câu trả lời là không tồn tại nhé:

f(f(x)) = x suy ra f là song ánh.

$x=1-(1-x)=f(f(1-x)+1)$ suy ra $f(x)=f(1-x)+1$, thay 1-x vào cái này ta có $f(1-x)=f(x)+1$ vô lý




#454173 Chứng minh rằng $x=a$

Đã gửi bởi tuan101293 on 30-09-2013 - 00:38 trong Số học

Bài này là IranMO 1998 có lời giải trong quyển number theory structure của titu nhé

http://blngcc.files....mber-theory.pdf




#454170 $x_1=a (a> -1); x_{n+1}=\frac{1}{1+x_n...

Đã gửi bởi tuan101293 on 30-09-2013 - 00:16 trong Dãy số - Giới hạn

đặt $$x_n = \frac{y_n}{z_n}$$ với $$y_1 = a, z_1 = 1$$

ta có công thức $$\frac{y_{n+1}}{z_{n+1}} = \frac{z_{n}}{y_{n}+z_{n}}$$

nên công thức truy hồi cho 2 dãy là $$y_{n+1} = z_{n} , z_{n+1} = y_{n}+z_{n}$$ hay $$y_{n+1} = z_{n}, z_{n+1} = z_{n} + z_{n-1}$$

nên $$z_{n}$$ là dãy Fibonacci và $$x_{n}$$ chính là tỷ lệ giữa 2 số Fibo liên tiếp nên dãy hội tụ và $$lim = \frac{\sqrt{5}-1}{2}$$

bài 2 thì tương tự 




#453945 phương trình $x^2+y^2+z^2=7^{2^n}$ có ít nhất $1...

Đã gửi bởi tuan101293 on 29-09-2013 - 17:42 trong Số học

ta quy nạp mệnh đề sau: ton tại bộ (x,y,z) thoả mãn điều kiện đề bài và có 2 số chẵn, 1 số lẻ.

với n = 1 ta có $$2^2+3^2+6^2 = 7^2$$

nếu có $$x^2+y^2+z^2 = 7^{2^n}$$, bộ $$(|x^2+y^2-z^2|,2xz,2yz)$$ thoả mãn (n+1).
thật vậy, lấy $$p|(x^2+y^2-z^2,2xz,2yz)$$ suy ra p lẻ
nếu $$p|z$$ suy ra $$p|x^2+y^2$$ suy ra $$p|x^2+y^2+z^2$$ suy ra p=7 vô lý
nếu z không chia hết cho p thì .... vô lý nốt
nên bộ 3 của n+1 nguyên tố cùng nhau. 
q.e.d



#307939 $$f(f(m)+f(n))=m+n,\forall m,n \in \mathbb{N^*}...

Đã gửi bởi tuan101293 on 03-04-2012 - 15:04 trong Phương trình hàm

+,Dễ thấy f đơn ánh và với mọi $n\ge 2$ thì tồn tại s để $f(s)=n$
+,thay (m+1,n-1) vào ta có :$f(m+1)-f(m)=f(n)-f(n-1)$ nên suy ra $f(m+1)-f(m)=f(2)-f(1)$
dễ thấy f(2)>f(1) nếu ko f(n)<f(1) (vô lý)
suy ra f(m+1)>f(m). xét tiếp số s mà f(s)=2. ta thấy nếu $s\ge 3$ suy ra f(2)<2 hay f(1)<1 loại nên ta có 2 TH
+,f(2)=2 suy ra f(n)=n
+,f(1)=2 suy ra thay (1,1) vào đề bài ta có f(2f(1))=2=f(1) hay f(1)=1/2, loại
suy ra f(n)=n



#307936 Ấn tượng khi thi HSG

Đã gửi bởi tuan101293 on 03-04-2012 - 14:47 trong Thi HSG cấp Tỉnh, Thành phố. Olympic 30-4. Đề thi và kiểm tra đội tuyển các cấp.

Mình cũng từng thi QG 2 phát lận. Lần nào thi cũng áp lực như nhau :(, vì mọi người đặt niềm tin lớn quá :D. Khi thi những kỳ thi 'to to ' thế này, hầu hết mọi người đều bảo thoải mái thì mới hết sức đc nhưng mình chả bao h làm đc thế, toàn làm đc đủ chỉ tiêu là nghỉ...Hơi tiếc nhưng đã qua rồi. Dù sao cũng là 1 kinh nghiệm quý báu @@



#307640 China TST 2012

Đã gửi bởi tuan101293 on 01-04-2012 - 21:23 trong Thi HSG Quốc gia và Quốc tế

Xét số $k$ bất kì có ước nguyên tố là $p_n$.
Nếu số $m$ là một số tốt,không chia hết cho $k$ và $m$ có ít nhất một ước nguyên tố $p_l$ với $l>n$ khi đó số $m.\frac{p_n}{p_l}<m$ và có số ước không ít hơn số ước của $m$. Dẫn đến mâu thuẫn.

mình không rõ đoạn này lắm, nếu $p_{n}$ là ước của $m$ thì sao ???



#307585 Tìm tất cả các hàm số $f:\mathbb{Z}\to\mathbb{Z}$ sa...

Đã gửi bởi tuan101293 on 01-04-2012 - 19:03 trong Phương trình hàm

Giả sử tồn tại $a,b$ sao cho $f(a)=f(b)$.
Cho $x=a,y=b$ và $x=b,y=a$ vào đề bài, ta có:
$$f(a+b+f(b))=f(a)+nb=f(a+b+f(a))=f(b)+na$$
suy ra $a=b$ hay $f$ la 1 đơn ánh,
Cho $y=0$ suy ra $f(0)=0$
Cho $x=-y$ suy ra: $f(f(y))=f(-y)+ny$, suy ra $ f(f(-y))=f(y)-ny$
Cho $x=f(-y)$ suy ra :
$$f(y)=f(f(-y))+ny=f(f(-y)+y+f(y))$$
Do $f$ đơn ánh suy ra $f(y)+f(-y)=0$ hay $f$ là hàm lẻ suy ra :
$$f(f(y))+f(y)=ny$$ (ngay trên)
Cho $y=f(y)$ suy ra :
$$f(x+f(y)+f(f(y)))=f(x)+nf(y)=f(x+ny)$$
Cho $x=0$ suy ra $f(ny)=nf(y)$, và thay $x=nx$ suy ra $f(x+y)=f(x)+f(y)$
suy ra $f(x)=xf(1)$ với mọi $x$ thuộc $\mathbb{Z}$
......(sory vi nha minh ko co vietkey)



#301616 Tồn tại vô số cặp số nguyên dương $(a;b)$

Đã gửi bởi tuan101293 on 29-02-2012 - 21:29 trong Số học

lâu lâu vào làm thử 1 bài @@
$a+b|ab+1$ nên $a+b|b(a+b)-ab-1=b^2-1$
tương tự ta có $a-b|b^2-1$, chú ý $(a+b,a-b)|2$ do (a,b)=1 suy ra $a^2-b^2|2(b^2-1)$
xài cái bất pt ở dưới, ta suy ra luôn phải chọn (a,b) để $a^2-b^2=2b^2-2$ hay là $a^2=3b^2-2$ (có nghiệm (1,1) nên hiển nhiên vô số nghiệm). Ta chỉ cần kiểm tra xem (a,b) có t/m 2 đk đầu ko
đến đây ta có a,b đồng dư mod 2 hay tồn tại (x,y) để
$a=x+y, b=x-y$ suy ra $x^2+y^2-4xy=1$ nên ta có ngay 2 đk ban đầu t/m
ĐPCM
(làm vội nên ko chắc)